LSAT and Law School Admissions Forum

Get expert LSAT preparation and law school admissions advice from PowerScore Test Preparation.

User avatar
 Dave Killoran
PowerScore Staff
  • PowerScore Staff
  • Posts: 5852
  • Joined: Mar 25, 2011
|
#44138
Complete Question Explanation
(The complete setup for this game can be found here: lsat/viewtopic.php?t=16592)

The correct answer choice is (D)


The question stem specifies that the associates are under examination.

As J cannot receive a majority of votes due to the first and third rules, answer choice (B) can be eliminated.

As O must be promoted during this year’s review, O cannot be on the associates roster, and thus answer choice (E) can be eliminated.

The remaining incorrect answer choices can be eliminated by examining the votes. O will not vote for G or T (or J, of course), and thus, for G or T to be promoted, G or T would need two votes total from R and H. While the combination of R’s and H’s votes promotes O, the total vote tally leaves G with one vote and T with no votes. Thus, neither can be promoted, and answer choices (A) and (C) (and (E)) can be eliminated. The correct answer choice is (D).
User avatar
 NegusAlfie
  • Posts: 6
  • Joined: Jan 22, 2021
|
#86892
This is a little confusing. If D is the correct answer, then it means that Wilford is promoted to associate. Per the rules, you can only secure promotion with a *majority* of the votes of seniors. Rafael votes for Willford, but Harrison never votes for him. We are not told whether Olivos votes for Willford. Meaning, we can only be certain of one vote. How then is he promoted to associate? Or, are we to construe 'majority' as including '1 of 2"?
 Robert Carroll
PowerScore Staff
  • PowerScore Staff
  • Posts: 1787
  • Joined: Dec 06, 2013
|
#86894
Negus,

That's a good point, but it's not an objection to the answer here. Let's see why.

The question says "...could be the complete roster of associates..." Note that question type: "could be". So we don't know whether Olivos will vote for Wilford. Thus, Wilford might get the following votes in the first year:

Promote Wilford: Rafael
Don't promote Wilford: Olivos, Harrison

But the following is also possible:

Promote Wilford: Rafael, Olivos
Don't promote Wilford: Harrison

We thus are unsure whether Wilford gets 1 out of 3 votes or 2 out of 3 votes. But 2 out of 3 is possible, right? And the question is asking what could be true. So, because Wilford COULD get 2/3 votes, Wilford COULD get a majority and be promoted. That's all we need for Wilford to be acceptable in an answer choice.

Robert Carroll
User avatar
 NegusAlfie
  • Posts: 6
  • Joined: Jan 22, 2021
|
#86918
Ah! Had glossed over the "could be true" nature of the question. Thanks for clarifying!
 kevwemarie@gmail.com
  • Posts: 4
  • Joined: Apr 10, 2023
|
#102562
I am confused as to why G was eliminated when it basically has the same condition as L, W and even S. They all have 1 vote while also having a "never vote for" condition.
What I mean by this is that;
for G, R votes for it while O never votes for it.
for L, H votes for it while R never votes for it.
for W, R votes for it while H never votes for it.
for S, H votes for it while R never votes for it.

Am I missing something? Help please.
User avatar
 Dave Killoran
PowerScore Staff
  • PowerScore Staff
  • Posts: 5852
  • Joined: Mar 25, 2011
|
#102587
Hi Kev,

I think that the nature of how this question is set up is confusing you. What we know from the question stem is the exact votes of R and H, and then beyond that we have the first rule involving who O will not vote for.

So with G specifically, we have R's vote for G from the question stem, and from the first rule we know O will not vote for G. Thus it looks like we have one for and one not for, right? But, from the question stem we also know H did not vote for G, since it was only L, O, and S who got votes from H. So, that means G can't get a majority, and thus answer choice (A) can be eliminated.

Please let me know if that helps. Thanks!

Get the most out of your LSAT Prep Plus subscription.

Analyze and track your performance with our Testing and Analytics Package.